Showing posts with label How to Prepare for Civil Services. Show all posts
Showing posts with label How to Prepare for Civil Services. Show all posts

Friday 29 November 2019

MISSION UPSC 17 TO 23 NOV. 2019 UPSC CURRENT AFF. TEST PAPER ENGLISH WITH ANSWER KEY

Image result for upsc images"



1.      Green crackers contain less amount of chemicals such as lithium, arsenic, barium and lead compared to regular crackers.
2.      Green crackers release water vapour and don’t allow the dust particles to rise.
3.      In India, green crackers are researched and developed by scientists at DRDO.

Which of the above statements is/are correct?

 a) 1, 2
 b) 2 only
 c) 2, 3
 d) 1, 3

Solution: b)
In October 2018, the Supreme Court allowed bursting of low-emission crackers relaxing the ‘complete ban’ issued in 2017. Such ‘green’ crackers were researched and developed by scientists at CSIR-NEERI as per the court’s directions.
‘Green’ crackers have a small shell size compared to traditional crackers. They are produced using less harmful raw materials and have additives which reduce emissions by suppressing dust.
Green crackers don’t contain banned chemicals such as lithium, arsenic, barium and lead. They are called Safe Water Releaser (SWAS), Safe Thermite Cracker (STAR) and Safe Minimal Aluminium (SAFAL) crackers.

Green crackers release water vapour and don’t allow the dust particles to rise. They are designed to have 30% less particulate matter pollution.

2.     Consider the following statements regarding International Treaty of Plant Genetic Resources for Food and Agriculture (ITPGRFA).

1.      It is a comprehensive international agreement for ensuring food security through the conservation, exchange and sustainable use of the world’s Plant Genetic Resources for Food and Agriculture.
2.      It aims to ensure that recipients share benefits they derive from the use of the plant genetic materials with the countries where they have been originated.
3.      It is not mandatory to ratify the treaty for accessing Plant Genetic materials.

Which of the above statements is/are correct?

 a) 1, 3
 b) 2, 3
 c) 1, 2
 d) 1, 2, 3

 Solution: c)
 The International Treaty on Plant Genetic Resources for Food and Agriculture was adopted by the Thirty-First Session of the Conference of the Food and Agriculture Organization of the United Nations on 3 November 2001.
It is also known as Seed Treaty as it is a comprehensive international agreement for ensuring food security through the conservation, exchange and sustainable use of the world’s Plant Genetic Resources for Food and Agriculture (PGRFA).
 The Treaty aims at:
recognizing the enormous contribution of farmers to the diversity of crops that feed the world;
establishing a global system to provide farmers, plant breeders and scientists with access to plant genetic materials;
ensuring that recipients share benefits they derive from the use of these genetic materials with the countries where they have been originated.
 Main Provisions:
Multilateral system: The treaty puts 64 of our most important crops – crops that together account for 80 percent of the food we derive from plants – into an easily accessible global pool of genetic resources that is freely available to potential users in the Treaty’s ratifying nations for some uses.
Access and benefit sharing: The Treaty facilitates access to the genetic materials of the 64 crops in the Multilateral System for research, breeding and training for food and agriculture. Those who access the materials must be from the Treaty’s ratifying nations and they must agree to use the materials totally for research, breeding and training for food and agriculture. The Treaty prevents the recipients of genetic resources from claiming intellectual property rights over those resources in the form in which they received them.

3.     Consider the following statements regarding Eurasian economic union.

1.      It is an international organization for regional economic integration.
2.      It has international legal personality and is established by the Treaty on the Eurasian Economic Union.
3.      Russia, Belarus, Armenia and Uzbekistan are the Member-States of the Eurasian Economic Union.

Which of the above statements is/are correct?

 a) 1, 3
 b) 1 only
 c) 2, 3
 d) 1, 2

Solution: d)
Eurasian economic union:
It is an international organization for regional economic integration.
It has international legal personality and is established by the Treaty on the Eurasian Economic Union.
Composition: Includes Russia, Belarus, Armenia, Kyrgyzstan and Kazakhstan.
It has free movement of goods, services and labour.
It has its own bureaucratic structure.

4.     Consider the following statements about Special 301 Report.

1.      It is prepared annually by the World Bank.
2.      The Report includes a list of “Priority Foreign Countries”, “Priority Watch List” and a “Watch List” countries which are judged on the basis of intellectual property laws.

Which of the above statements is/are correct?

 (a) 1 only
 (b) 2 only
 (c) Both 1 and 2
 (d) Neither 1 nor 2

Solution: b)
The Special 301 Report is prepared annually by the Office of the United States Trade Representative (USTR) that identifies trade barriers to United States companies and products due to the intellectual property laws, such as copyright, patents and trademarks, in other countries.
The annual Special 301 Report includes a list of “Priority Foreign Countries”, that are judged to have inadequate intellectual property laws; these countries may be subject to sanctions. In addition, the report contains a “Priority Watch List” and a “Watch List”, containing countries whose intellectual property regimes are deemed of concern.

5.     Consider the following statements regarding Marawah Island, recently seen in news

1.      Marawah Island is located off the coast of Abu Dhabi.
2.      The oldest known natural pearl in the world has been discovered on Marawah Island.

Which of the above statements is/are correct?

 a) 1 only
 b) 2 only
 c) Both
 d) None

Solution: c)
The oldest known natural pearl in the world has been discovered by Abu Dhabi archaeologists working at a Neolithic site on Marawah Island, just off the coast of Abu Dhabi.
Dubbed the ‘Abu Dhabi Pearl’, it was found in layers that have been radiocarbon dated to 5,800-5,600 BCE, during the Neolithic period.


6. Consider the following statements
regarding US-India Defence Technology and Trade Initiative.

1.                  It is a mechanism to make sure that senior leaders from both countries are engaged consistently to strengthen the opportunities in the field of defence.
2.                  It aims to strengthen India’s defence industrial base, exploring new areas of technological development and expanding U.S.-India business ties.

Which of the above statements is/are correct?

a) 1 only
b) 2 only
 c) Both
 d) None

Solution: c)
 In August 2018, the US granted to India the designation of Strategic Trade Authority Tier 1 or STA-1, “providing India with greater supply-chain efficiency by allowing US companies to export a greater range of dual-use and high-technology items to India under streamlined processes.” This authorisation is the equivalent of NATO allying with Japan, South Korea and Australia.
 DTTI came about to expedite the scope of cooperation on defence technology that become narrow due to the presence of differing bureaucratic processes and legal requirements.
 While DTTI is not a treaty or law, it is a flexible mechanism to make sure that senior leaders from both countries are engaged consistently to strengthen the opportunities in the field of defence. Its central aims include strengthening India’s defence industrial base, exploring new areas of technological development and expanding U.S.-India business ties.

7. Consider the following statements regarding Eco-sensitive zones.

1.                  Eco-Sensitive Zones (ESZs) are areas notified by the Ministry of Environment, Forests and Climate Change (MoEFCC) around Protected Areas, National Parks and Wildlife Sanctuaries.
2.                  They act as a transition zone from areas of high protection to areas involving lesser protection.
3.                  Eco-Sensitive Zones are defined and mentioned in Environment (Protection) Act, 1986.

Which of the above statements is/ are correct?

a) 1, 3
 b) 2, 3
 c) 1, 2
 d) 1, 2, 3

Solution: c)
 Eco-Sensitive Zones (ESZs) or Ecologically Fragile Areas (EFAs) are areas notified by the Ministry of Environment, Forests and Climate Change (MoEFCC), Government of India around Protected Areas, National Parks and Wildlife Sanctuaries. The purpose of declaring ESZs is to create some kind of “shock absorbers” to the protected areas by regulating and managing the activities around such areas. They also act as a transition zone from areas of high protection to areas involving lesser protection.
ESZ are regulated by central government through Min. of Environment, Forests and Climate change (MoEFCC). Ministry came out with new guidelines for regulation of such areas in 2011.
The Environment Protection Act, 1986 does not mention the word “Eco-sensitive Zones”.
Image result for upsc images"
8. Consider the following statements regarding Kartarpur Sahib pilgrim corridor.

1.      It is a visa-free border crossing and secure corridor, connecting the Gurdwara Darbar Sahib in Pakistan to the border with India.
2.      The gurdwara in Kartarpur stands on the bank of the river Ravi.
3.      The place holds significance because the first guru of Sikhism, Guru Nanak was born in Kartarpur.

Which of the above statements is/are correct?

a) 1, 2
 b) 1 only
 c) 2, 3
 d) 1, 3

Solution: a)

The Kartarpur Corridor is a visa-free border crossing and secure corridor, connecting the Gurdwara Darbar Sahib in Pakistan to the border with India.
7.         The gurdwara in Kartarpur stands on the bank of the Ravi, about 120 km northeast of Lahore.
8.         It was here that Guru Nanak assembled a Sikh community and lived for 18 years until his death in 1539.

9. Consider the following statements about International Maritime Organization (IMO).

1.  It is independent to United Nations.
2.         It is the global standard-setting authority for the safety, security and environmental performance of international shipping.
3.         India was re-elected as Member to the Council of the International Maritime Organization for two years (2018-19).

Which of the above statements is/are correct?

a) 1, 2
b) 1, 3
c) 2, 3
d) 1, 2, 3

Solution: c)

The International Maritime Organization – is the United Nations specialized agency with responsibility for the safety and security of shipping and the prevention of marine pollution by ships.
The IMO’s primary purpose is to develop and maintain a comprehensive regulatory framework for shipping and its remit today includes safety, environmental concerns, legal matters, technical co-operation, maritime security and the efficiency of shipping.
 India was re-elected as Member of International Maritime Council for two years (2018-19)

10. Consider the following statements regarding Tech Sagar.

1.      TechSagar is a consolidated and comprehensive repository of India’s cyber tech capabilities.
2.      The National Cyber Security Coordinator’s office in partnership with Data Security Council (DSCI) of India launched TechSagar.
3.      Data Security Council (DSCI) is a not-for-profit, industry body on data protection in India.

Which of the above statements is/are correct?

a) 1, 2
b) 1, 3
c) 2, 3
d) 1, 2, 3

Solution: d)
 The National Cyber Security Coordinator’s office in partnership with Data Security Council (DSCI) of India launched TechSagar – a platform to discover India’s technological capability through a portal. The portal will list business and research entities from the IT industry, startups, academia, and individual researchers.
 As India aspires to become a trillion-dollar digital economy, the repository will facilitate new opportunities for businesses and academia to collaborate, connect and innovate in future.
 TechSagar is a consolidated and comprehensive repository of India’s cyber tech capabilities which provides actionable insights about capabilities of the Indian Industry, academia and research across 25 technology areas like internet of things (IoT), Artificial Intelligence (AI), Machine Learning (ML), blockchain, cloud & virtualisation, robotics & automation, ar/vr, wireless & networking, and more.
 DSCI is a not-for-profit, industry body on data protection in India, setup by NASSCOM.

11. Consider the following statements regarding Cape Town Agreement, sometimes seen in news.

1.      Cape Town Agreement was adopted by the United Nations Convention on the Law of the Sea (UNCLOS) to help combat illegal, unregulated and unreported (IUU) fishing.
2.      It seeks to introduce mandatory safety measures for fishing vessels.
3.      India was the first country to sign and ratify the agreement.

Which of the above statements is/are correct?

a) 1, 3
b) 2 only
c) 1, 2
d) 2, 3

Solution: b)
 India, ranked third in fisheries, is yet to ratify a global regulatory regime adopted by the International Maritime Organization (IMO) for safety of fishing vessels.
 Cape Town Agreement seeks to introduce mandatory safety measures for fishing vessels of 24 metres and over in length.
 The Cape Town Agreement was adopted by the IMO in 2012 to help combat illegal, unregulated and unreported (IUU) fishing.
The treaty will enter into force 12 months after at least 22 nations, with an aggregate 3,600 fishing vessels of 24 metres and over in length operating on the high seas, have expressed their consent to be bound by it.
The Cape Town Agreement includes mandatory international requirements for stability and associated seaworthiness, machinery and electrical installations, life-saving appliances, communications equipment and fire protection, as well as fishing vessel construction. It is aimed at facilitating better control of fishing vessel safety by flag, port and coastal states.

12. Consider the following statements regarding Cape Town Agreement, sometimes seen in news.

1.      Cape Town Agreement was adopted by the United Nations Convention on the Law of the Sea (UNCLOS) to help combat illegal, unregulated and unreported (IUU) fishing.
2.      It seeks to introduce mandatory safety measures for fishing vessels.
3.      India was the first country to sign and ratify the agreement.

Which of the above statements is/are correct?

a) 1, 3
b) 2 only
c) 1, 2
d) 2, 3

Solution: b)

13. Consider the following statements regarding Cape Town Agreement, sometimes seen in news.

1.      Cape Town Agreement was adopted by the United Nations Convention on the Law of the Sea (UNCLOS) to help combat illegal, unregulated and unreported (IUU) fishing.
2.      It seeks to introduce mandatory safety measures for fishing vessels.
3.      India was the first country to sign and ratify the agreement.

Which of the above statements is/are correct?

a) 1, 3
b) 2 only
c) 1, 2
d) 2, 3

Solution: b)

14. Consider the following statements regarding Jal Jeevan Mission.

1.      The chief objective of the Mission is to revive traditional water sources in all rural areas by 2024.
2.      It also aims to create local infrastructure for rainwater harvesting, groundwater recharge and management of household waste water for reuse in agriculture.
3.      The Jal Jeevan Mission will converge with other Central and State Government Schemes to achieve its objectives of sustainable water supply management.

Which of the above statements is/are correct?

a) 1, 2
b) 1, 3
c) 2, 3
d) 1, 2, 3

Solution: c)
 Jal Jeevan Mission:
The Mission was announced in August 2019.
The chief objective of the Mission is to provide piped water supply (Har Ghar Jal) to all rural and urban households by 2024.
It also aims to create local infrastructure for rainwater harvesting, groundwater recharge and management of household waste water for reuse in agriculture.
The Jal Jeevan Mission is set to be based on various water conservation efforts like point recharge, desilting of minor irrigation tanks, use of greywater for agriculture and source sustainability.
The Jal Jeevan Mission will converge with other Central and State Government Schemes to achieve its objectives of sustainable water supply management across the country.

15.      Consider the following statements regarding District Mineral Foundation (DMF).

1.      District MineralFoundation (DMF) was instituted under the Mines and Minerals (Development and Regulation) (MMDR) Amendment Act 2015.
2.      District Mineral Foundation (DMF) is a trust set up as a non-profit body, in all districts to work for the interest and benefit of persons and areas affected by mining related operations.
3.      It is funded through the contributions from miners.
4.      The various state DMF rules and the Pradhan Mantri Khanij Khestra Kalyan Yojana (PMKKKY) guidelines stipulate Education, Health and Livelihood and skill development as “high priority” issues for DMFs.

Which of the above statements is/are correct?

a) 1, 2, 3
b) 2, 3, 4
c) 1, 3, 4
d) 1, 2, 3, 4

Solution: c)
DMFs were instituted under the Mines and Minerals (Development and Regulation) (MMDR) Amendment Act 2015.
District Mineral Foundation (DMF) is a trust set up as a non-profit body, in those districts affected by the mining works, to work for the interest and benefit of persons and areas affected by mining related operations. It is funded through the contributions from miners.
Jurisdiction: Its manner of operation comes under the jurisdiction of the relevant State Government.
 The various state DMF rules and the Pradhan Mantri Khanij Khestra Kalyan Yojana (PMKKKY) guidelines stipulate some “high priority” issues for DMFs, including:
11.       Drinking water.
12.       Health
13.       Women and child welfare.
14.       Education
15.       Livelihood and skill development.
16.       Welfare of aged and disabled.
17.       Sanitation
 Pradhan Mantri Khanij Kshetra Kalyan Yojana (PMKKKY):
The programme is meant to provide for the welfare of areas and people affected by mining related operations, using the funds generated by District Mineral Foundations (DMFs).
Source

16.      Consider the following statements about Black Holes

1. Most Black holes are formed from the remnants of a large star that dies in a supernova explosion.
2. Black holes were predicted by Einstein’s theory of general relativity.
3. Scientists can directly observe black holes with telescopes that detect x-rays, light, or other forms of electromagnetic radiation.

Which of the above statements is/are correct?

a) 1, 2
b) 1, 3
c) 2 only
d) 2, 3

Solution: a)
 The idea of an object in space so massive and dense that light could not escape it has been around for centuries. Most famously, black holes were predicted by Einstein’s theory of general relativity, which showed that when a massive star dies, it leaves behind a small, dense remnant core.
 Scientists can’t directly observe black holes with telescopes that detect x-rays, light, or other forms of electromagnetic radiation. We can, however, infer the presence of black holes and study them by detecting their effect on other matter nearby. If a black hole passes through a cloud of interstellar matter, for example, it will draw matter inward in a process known as accretion. A similar process can occur if a normal star passes close to a black hole. In this case, the black hole can tear the star apart as it pulls it toward itself. As the attracted matter accelerates and heats up, it emits x-rays that radiate into space. Recent discoveries offer some tantalizing evidence that black holes have a dramatic influence on the neighborhoods around them – emitting powerful gamma ray bursts, devouring nearby stars, and spurring the growth of new stars in some areas while stalling it in others.
 Most black holes form from the remnants of a large star that dies in a supernova explosion.


17. Consider the following statements regarding Solar PV cells.

1.      Pradhan Mantri Kisan Urja Suraksha evam Utthan Mahabhiyan (PM-KUSUM) scheme mandate the use of domestically manufactured solar PV cells.
2.      A solar PV cell shall be considered to be domestically manufactured only if the same has been manufactured in India using un-diffused silicon wafers.

Which of the above statements is/are correct?

a) 1 only
b) 2 only
c) Both
d) None

Solution: c)
 The Ministry of New and Renewable Energy (MNRE) has recently issued a clarification stating that solar PV cells manufactured using diffused silicon wafers or blue wafers shall not be considered domestically manufactured solar PV cells, and hence, would not qualify for MNRE’s schemes/programmes. This decision can be expected to significantly reduce the country’s dependence on foreign imports and establish India as a prominent manufacturing hub of solar products. The move can also help in reducing initial costs associated with solar PV systems and improve its adoption in the country.
MNRE’s flagship programmes such as the Pradhan Mantri Kisan Urja Suraksha evam Utthan Mahabhiyan (PM-KUSUM) scheme mandate the use of domestically manufactured solar PV cells. Despite the mandate, it was noticed that some manufacturers were using imported semi-processed PV cells or blue wafers as raw materials to manufacture solar PV cells.
MNRE stated that “A solar PV cell shall be considered to be domestically manufactured only if the same has been manufactured in India using un-diffused silicon wafers (generally called ‘black wafers’).”

18. Consider the following statements regarding Global Snow Leopard and Ecosystem Protection Program (GSLEP).

1.      The GSLEP is a world first joint initiative that aims to conserve the snow leopard within the broader context of also conserving valuable high mountain ecosystems.
2.      Snow leopard range countries signed Bishkek Declaration to work towards the goal of the GSLEP.
3.      Every year India hosts the GSLEP meet under the aegis of Ministry of environment, forest and climate change.

Which of the above statements is/are correct?

a) 1, 3
b) 2, 3
c) 1, 2
d) 1, 2, 3

Solution: c)
 The GSLEP is a world first joint initiative that aims to conserve the  snow leopard within the broader context of also conserving valuable high mountain ecosystems.
GSLEP unites all 12 range country governments, nongovernmental and inter-governmental organisations, local communities, and the private sector around this aim.
In 2013 the 12 snow leopard range countries and partners signed the Bishkek Declaration (in the Kyrgyz Republic capital city) and agreed to the goal of the GSLEP for the 7 years through 2020. The snow leopard range countries agree, with support from stakeholder and interested organisations, to work together to identify and secure at least 20 snow leopard landscapes across the cat’s range by 2020 or, in short – “Secure 20 by 2020.”
 The 4th steering committee meeting of the Global Snow Leopard and Ecosystem Protection Program (GSLEP) was held recently in New Delhi, where environment minister Prakash Javadekar launched the national protocol for snow leopard population assessment in India.
This is for the first time that India hosted the GSLEP meet, under the aegis of ministry of environment, forest and climate change (MoEF).

19. Consider the following statements regarding Snow Leopard.

1.      Snow leopard is listed as Critically endangered under the IUCN Red List.
2.      In India, the snow leopard habitat is spread over the northern Himalayan mountains of Jammu and Kashmir, Himachal Pradesh, Uttarakhand, Arunachal Pradesh and Sikkim.
3.      The Union Government had launched SECURE Himalaya project for the protection of Snow Leopard.

Which of the above statements is/are correct?

a) 1, 2
b) 1, 3
c) 2, 3
d) 1, 2, 3

Solution: c)
 Snow leopard is listed as Vulnerable under the IUCN Red List. In India, the snow leopard habitat is spread over the northern Himalayan mountains of Jammu and Kashmir, Himachal Pradesh, Uttarakhand, Arunachal Pradesh and Sikkim.
 The Union Government had launched SECURE Himalaya, a six-year project to ensure conservation of locally and globally significant biodiversity, land and forest resources in high Himalayan ecosystem spread over four states viz. Himachal Pradesh, Jammu and Kashmir, Uttarakhand and Sikkim.
It was launched by Union Ministry of Environment, Forests and Climate Change (MoEFCC) in association with the United Nations Development Programme (UNDP).
Protection of snow leopard and other endangered species and their habitats is one of the key components of the project which will also focus on securing livelihoods of the people in the region and enhancing enforcement to reduce wildlife crime.

20. Consider the following statements regarding President’s Rule in India.

1.      President’s Rule is imposed when the government of the state cannot be carried on in accordance with the provisions of the Constitution.
2.      The imposition of the President’s rule is approved by the president and does not require the sanction of the Parliament.
3.      A proclamation of President’s Rule may be revoked by the President at any time by a subsequent proclamation without parliamentary approval.

Which of the above statements is/are correct?

a) 1 only
b) 2, 3
c) 1, 3
d) 1, 2

Solution: c)
 Article 356 of the Constitution of India gives President of India the power to suspend state government and impose President’s rule of any state in the country if “if he is satisfied that a situation has arisen in which the government of the state cannot be carried on in accordance with the provisions of the Constitution”.
Upon the imposition of this rule, there would be no Council of Ministers. The Vidhan Sabha is either dissolved or prorogued.
The state will fall under the direct control of the Union government, and the Governor will continue to be head the proceedings, representing the President of India – who is the Head of the State.
The imposition of the President’s rule requires the sanction of both the houses of Parliament.
If approved, it can go on for a period of six months. However, the imposition cannot be extended for more than three years, and needs to be brought before the two houses every six months for approval.

A proclamation of President’s Rule may be revoked by the President at any time by a subsequent proclamation. Such a proclamation does not require parliamentary approval.

21. Consider the following statements about National Mineral Policy 2019.

1.                  Encourages the private sector to take up exploration.
2.                  It proposes to grant status of industry to mining activity.
3.                  The policy introduced the concept of Inter-Generational Equity that deals with the well-being not only of the present generation but also of the generations to come.

Which of the above statements is/are correct?

a) 1, 2
b) 2, 3
c) 1, 3
 d) 1, 2, 3

Solution: d)
 The National Mineral Policy 2019 includes provisions which will give boost to mining sector such as
introduction of Right of First Refusal for RP/PL holders,
encouraging the private sector to take up exploration,
auctioning in virgin areas for composite RP cum PL cum ML on revenue share basis,
encouragement of merger and acquisition of mining entities and
transfer of mining leases and creation of dedicated mineral corridors to boost private sector mining areas.
The 2019 Policy proposes to grant status of industry to mining activity to boost financing of mining for private sector and for acquisitions of mineral assets in other countries by private sector
It also mentions that Long-term import-export policy for mineral will help private sector in better planning and stability in business
The Policy also mentions rationalize reserved areas given to PSUs which have not been used and to put these areas to auction, which will give more opportunity to private sector for participation
The Policy also mentions to make efforts to harmonize taxes, levies & royalty with world benchmarks to help private sector
 NMP 2019 aims to attract private investment through incentives while the efforts would be made to maintain a database of mineral resources and tenements under mining tenement systems.   The new policy focusses on use coastal waterways and inland shipping for evacuation and transportation of minerals and encourages dedicated mineral corridors to facilitate the transportation of minerals.  The utilization of the district mineral fund for equitable development of project affected persons and areas. NMP 2019 proposes a long-term export-import policy for the mineral sector to provide stability and as an incentive for investing in large scale commercial mining activity.
The 2019 Policy also introduces the concept of Inter-Generational Equity that deals with the well-being not only of the present generation but also of the generations to come and also proposes to constitute an inter-ministerial body to institutionalize the mechanism for ensuring sustainable development in mining.

22. Consider the following statements regarding Contract farming.

1.                  Contract farming refers to an agreement between farmers and marketing firms for the production and supply of agricultural products under forward agreements, frequently at predetermined prices.
2.                  In India, contract farming is regulated under the Agricultural Produce Market Committees (Development and Regulation) (APMC) Act of 2003.
3.                  Karnataka has become the first State in the country to enact a law on contract farming.

Which of the above statements is/ are correct?

a) 1, 2
 b) 1, 3
 c) 2, 3
d) 1, 2, 3

Solution: a)
 Contract farming refers to an agreement between farmers and marketing firms for the production and supply of agricultural products under forward agreements, frequently at predetermined prices.
 At present, contract farming is regulated under the Agricultural Produce Market Committees (Development and Regulation) (APMC) Act of 2003. This is the law that legalises contract farming.
 Tamil Nadu has become the first State in the country to enact a law on contract farming with President Ram Nath Kovind giving assent to the Agricultural Produce and Livestock Contract Farming and Services (Promotion and Facilitation) Act.

23. Consider the following statements.

1.      The absentee voter refers to a vote cast by someone who is unable to go to the polling station.
2.      The power to include any category of persons like disabled and people over 80 years of age in the absentee voter list lies with the Election Commission of India.

Which of the above statements is/are correct?

 a) 1 only
 b) 2 only
 c) Both
 d) None

Solution: a)
The disabled and people over 80 years of age can now cast their vote through postal ballot. At present, voting through postal ballot is available to armed forces and those assigned poll duties.
On the recommendation of the Election Commission, the Ministry of Law and Justice has amended the Conduct of Election Rules, 1961, allowing senior citizens and person with disabilities in the absentee voter list.
The absentee voter refers to a vote cast by someone who is unable to go to the polling station.

24. Consider the following statements regarding Pardoning powers of President.

1.      The power of pardon shall be exercised by the President on the advice of Council of Ministers.
2.      The President can exercise these powers in all cases where the punishment or sentence is for an offence against any law relating to a matter to which the executive power of the Union extends.
3.      The constitution has provided a small window for judicial review of the pardon powers of President for the purpose of ruling out any arbitrariness.

Which of the above statements is/are correct?

 a) 1, 3
 b) 1, 2
 c) 2, 3
 d) 1, 2, 3

Solution: b)
The President can exercise these powers:
In all cases where the punishment or sentence is by a court martial;
In all cases where the punishment or sentence is for an offence against any law relating to a matter to which the executive power of the Union extends;
In all cases where the sentence is a sentence of death.
 This power of pardon shall be exercised by the President on the advice of Council of Ministers.
The constitution does not provide for any mechanism to question the legality of decisions of President or governors exercising mercy jurisdiction.
But the SC in Epuru Sudhakar case has given a small window for judicial review of the pardon powers of President and governors for the purpose of ruling out any arbitrariness.

25. Consider the following statements about United Nations Economic and Social Commission for Asia and the Pacific (ESCAP).

1.      It is the regional development arm of the United Nations for the Asia-Pacific region, established in 2000.
2.      It is the largest United Nations body serving the Asia-Pacific region.
3.      India is the founder member of UNESCAP.

Which of the above statements is/are correct?

 a) 1, 2
 b) 2, 3
 c) 1, 3
 d) 2 only

Solution: b)
The United Nations Economic and Social Commission for Asia and the Pacific (ESCAP) is the regional development arm of the United Nations for the Asia-Pacific region.
ESCAP is the most comprehensive of the United Nations five regional commissions, and the largest United Nations body serving the Asia-Pacific region.
Established in 1947 with its headquarters in Bangkok, Thailand, ESCAP works to overcome some of the region’s greatest challenges by providing results-oriented projects, technical assistance and capacity building to member States.
India, founder member of UNESCAP, has been closely involved and played a prominent role in its deliberations from its initial years.

26. Consider the following statements regarding National Pension System (NPS).

1.      National Pension System (NPS) is a pension cum investment scheme launched by Government of India to provide old age security to Citizens of India.
2.      The Scheme is regulated by Pension Fund Regulatory and Development Authority (PFRDA).
3.      Non-Resident Indians (NRIs) and Overseas Citizen of India (OCIs) are not eligible to invest in NPS.

Which of the above statements is/are correct?

 a) 1, 3
 b) 2, 3
 c) 1, 2
 d) 1, 2, 3

Solution: c)
National Pension System (NPS) is a pension cum investment scheme launched by Government of India to provide old age security to Citizens of India. It brings an attractive long-term saving avenue to effectively plan your retirement through safe and regulated market-based return. The Scheme is regulated by Pension Fund Regulatory and Development Authority (PFRDA). National Pension System Trust (NPST) established by PFRDA is the registered owner of all assets under NPS.
The Pension Fund Regulatory and Development Authority (PFRDA) has added one more category of investors who can invest in the National Pension System. PFRDA has stated that now Overseas Citizen of India (OCIs) can enrol to invest in NPS tier-1 accounts.

27. Consider the following statements regarding Overseas Indian Citizen.

1.      A foreign national, who was eligible to become citizen of India on 26.01.1950 is eligible for registration as Overseas Citizen of India (OCI).
2.      If the applicant had ever been a citizen of Pakistan or Bangladesh, he/she will not be eligible for OCI.

Which of the above statements is/are incorrect?

 (a) 1 only
 (b) 2 only
 (c) Both 1 and 2
 (d) Neither 1 nor 2

Solution: d)
According to Ministry of External Affairs website, an OCI is “A foreign national, who was eligible to become citizen of India on 26.01.1950 or was a citizen of India on or at any time after 26.01.1950 or belonged to a territory that became part of India after 15.08.1947 is eligible for registration as Overseas Citizen of India (OCI). Minor children of such person are also eligible for OCI. However, if the applicant had ever been a citizen of Pakistan or Bangladesh, he/she will not be eligible for OCI.”

28. Which of the following statements are correct with respect to the Narmada River?

1.      Narmada rises from the Amarkantak Plateau region in Madhya Pradesh.
2.      The river flows in a rift valley, flowing west between the Satpura and Vindhya mountain ranges.
3.      The river’s basin extends over the states of Madhya Pradesh and Gujarat only.

Options:

a. 1 and 2 only
b. 2 and 3 only
c. 1 and 3 only
d. 1, 2 and 3

Answer: a
Explanation:
The Narmada, the largest west flowing river of the Peninsula, rises near Amarkantak range of mountains in Madhya Pradesh. It is the fifth-largest river in the country and the largest one in Gujarat. It traverses through Madhya Pradesh, Maharashtra and Gujarat and meets the Gulf of Cambay.


29. Which of the following species is not classified as critically endangered in India?

a. Rameshwaram parachute spider
b. Namdapha flying squirrel
c. Pygmy hog
d. Lion-tailed macaque

Answer: d
Explanation:
Some examples of critically endangered species in India:
Rameshwaram parachute spider, Namdapha flying squirrel, Pygmy hog, Great Indian bustard, Bengal florican, Jerdon’s courser, Ganges shark, Pygmy Hog Sucking Louse, Northern river terrapin, Gharial, Chinese pangolin, Kondana soft-furred rat, Malabar large-spotted civet, Kashmir Stag.
Some examples of endangered species in India:
Red panda, Nilgiri tahr, Sangai, Asiatic lion, Bengal tiger and Lion-tailed macaque.
Note the list is not exhaustive.

30. Which of the following pairs are not correctly matched?

a. Madhubani Paintings: Bihar
b. Chanderi Sarees: Uttar Pradesh
c. Bidriware: Maharashtra
d. Gobindabhog Rice: West Bengal

Answer: c
Explanation:
Bidriware is associated with the state of Karnataka.
Bidriware is a metal handicraft from Bidar. It was developed in the 14th century C.E. during the rule of the Bahamani Sultans. The term ‘Bidriware’ originates from the township of Bidar, which is still the chief centre for the manufacture of the unique metalware. Due to its striking inlay artwork, Bidriware is an important export handicraft of India and is prized as a symbol of wealth. The metal used is a blackened alloy of zinc and copper inlaid with thin sheets of pure silver. This native art form has obtained Geographical Indications (GI) tag.

31. Which of the following statements is/are correct?

1.      In India, more than 50% of the land is under cultivation.
2.      The area under forest cover in India is about 30%.

Options:

a. 1 only
b. 2 only
c. Both 1 and 2
d. Neither 1 nor 2

Answer: a
Explanation:
In India, about 51.09% of the land is under cultivation, 21.81% under forest and 3.92% under pasture.

32. Which of the following are incorrectly matched?

a. Red Panda: Nokrek Biosphere Reserve
b. Dugong: Sunderbans
c. Giant Squirrel: Panchmarhi Biosphere Reserve
d. Snow Leopard: Khangchendzonga Biosphere Reserve

Answer: b
Explanation:
Dugong is found mainly in the Gulf of Mannar Biosphere Reserve.

33. Which of the following statements is/are correct?

1.      The maximum gap between two sessions of Parliament cannot be more than six months.
2.      Prorogation means the termination of a session of the House by an order made by the President under article 85(2)(a) of the Constitution.

The President can also prorogue the House while in session.

Options:

a. 1 only
b. 2 only
c. 1 and 2 only
d. 1, 2 and 3

Answer: d
Explanation:
The maximum gap between two sessions of Parliament cannot be more than six months. In other words, the Parliament should meet at least twice a year.
Prorogation means the termination of a session of the House by an order made by the President under article 85(2)(a) of the Constitution. Prorogation terminates both the sitting and session of the House. Usually, within a few days after the House is adjourned sine die by the presiding officer, the President issues a notification for the prorogation of the session. However, the President can also prorogue the House while in session.

34. With reference to the cultivation of crops in India, consider the following statements:

1.      India is the largest producer of pulses in the world.
2.      India is the largest importer of pulses in the world.
3.      Gram dal is the most grown pulse in India.

Which of the given statement/s is/are correct?

a. 2 only
b. 2 and 3 only
c. 1, 2 and 3
d. 1 and 3 only

Answer: c
Explanation:
India is the largest producer (25% of global production), consumer (27% of world consumption) and importer (14%) of pulses in the world. Pulses account for around 20 per cent of the area under foodgrains and contribute around 7-10 per cent of the total foodgrain production in the country. Though pulses are grown in both Kharif and Rabi seasons, Rabi pulses contribute more than 60 per cent of the total production.
Gram is the most dominant pulse having a share of around 40 per cent in the total production followed by Tur/Arhar at 15 to 20 per cent and Urad/Black Matpe and Moong at around 8-10 per cent each. Madhya Pradesh, Maharashtra, Rajasthan, Uttar Pradesh and Karnataka are the top five pulses producing States.

35. Jayakwadi dam is located on the river -
a. Godavari
b. Tungabhadra
c. Mahanadi
d. Brahmaputra

Answer: a

36. Which of the following is/are included in the list of UNESCO World Heritage Sites:

1.      Rani ki Vav
2.      Modhera Sun Temple
3.      Champaner-Pavagadh Archaeological Park

Choose the correct option:

a. 1 and 2 only
b. 1 only
c. 1 and 3 only
d. 2 only

Answer: c
Explanation:
Champaner-Pavagadh Archaeological Park, a UNESCO World Heritage Site, is located in Panchmahal district in Gujarat. Located on the banks of the historical and mythical river Saraswati in Patan, Rani ni Vav is the latest addition to the list from Gujarat. Built in the 11th Century, the step-well was originally a memorial to the king, and slowly took up the name of the queen.

37. Which of the following is/are the applications of Cartosat-2?

1.      Detailed mapping
2.      Geographical Information System (GIS) applications
3.      Land Information System (LIS) applications

Choose the correct option:

a. 1 only
b. 1 and 2 only
c. 1 and 3 only
d. 1, 2 and 3

Answer: d
Explanation:
Cartosat-2 is an Earth Observation satellite launched on PSLV-C40/Cartosat-2 Series Satellite Mission. The imagery sent by Cartosat-2 satellite has the following applications. It is useful for cartographic applications (detailed mapping), urban and rural applications, coastal land use and regulation, utility management like road network monitoring, water distribution, creation of land use maps, change detection to bring out geographical and manmade features and various other Land Information System (LIS) as well as Geographical Information System (GIS) applications.

38. Consider the following statements with respect to Pradhan Mantri Matru Vandana Yojana:
1.      It is a centrally sponsored scheme being executed by the Ministry of Women and Child Development.
2.      Under the programme, eligible pregnant women are entitled for cash assistance irrespective of the age of mother and number of children.

Choose the correct option:

a. 1 only
b. 2 only
c. Both 1 and 2
d. Neither 1 nor 2

Answer: a
Explanation:
Pradhan Mantri Matru Vandana Yojana is a centrally sponsored scheme being executed by the Ministry of Women and Child Development.
Target beneficiaries for the scheme are:
All Pregnant Women and Lactating Mothers, excluding those who are in regular employment with the Central Government or the State Governments or PSUs or those who are in receipt of similar benefits under any law for the time being in force.
All eligible Pregnant Women and Lactating Mothers who have their pregnancy on or after 01.01.2017 for the first child only.

39. Consider the following statements about Olive Ridley Turtles:

1.      Hope Island of Coringa Wildlife Sanctuary of Andhra Pradesh is one of the major breeding grounds for these turtles.
2.      Olive Ridley turtles feed on algae and seaweed only.
3.      They are classified as Critically Endangered as per the IUCN Red List.

Which of the given statement/s is/are incorrect?

a. 1 only
b. 2 only
c. 1 and 2 only
d. 2 and 3 only

Answer: d
Explanation:
Olive Ridley sea turtles come in large numbers for nesting to Odisha coasts. The mass nesting draws worldwide attention to the conservation of Olive Ridley in Odisha. The synchronised nesting in mass numbers is called Arribadas. They are mostly carnivorous and feed on jellyfish, snails, crabs, and shrimp. They occasionally consume algae and seaweed. The major breeding ground for these turtles is Rushikulya (Odisha), Dharma (Odisha), Devi estuary (Odisha), Astaranga Coast (Odisha), Gahirmatha beach (Odisha) and Hope Island of Coringa Wildlife Sanctuary (Andhra Pradesh). They are classified as Vulnerable — IUCN Red List.

40. Interferometric Synthetic Aperture Radar (InSAR) is a radar technique used in

1.      Geodesy
2.      Remote sensing
3.      Mapping ground deformation

Which of the given statement/s is/are correct?

a. 1 only
b. 1 and 3 only
c. 1, 2 and 3 only
d. 3 only

Answer: c
Explanation:
Interferometric synthetic aperture radar, abbreviated InSAR is a radar technique used in geodesy and remote sensing. This geodetic method uses two or more synthetic aperture radar (SAR) images to generate maps of surface deformation or digital elevation, using differences in the phase of the waves returning to the satellite. The technique is also used for mapping ground deformation using radar images of the Earth’s surface that are collected from orbiting satellites. Unlike visible or infrared light, radar waves penetrate most weather clouds and are equally effective in darkness. InSAR helps in generating maps of how a place would look after an earthquake, or a detonation.

41. “Ghamr” and “Baqura” in news are territories in conflict between which two countries?

a. UAE-Saudi Arabia
b. Israel – Jordan
c. Turkey-Syria
d. Pakistan-Afghanistan

Answer: b
Explanation:
Israel - Jordan Border

42. Which of the following is/are correctly matched?

1.      Indo-Tibetan Border Police (ITBP) -China
2.      Border Security Force – Pakistan
3.      Sashastra Seema Bal – Bangladesh

Choose the correct option:

a. 1 and 2 only
b. 2 and 3 only
c. 3 only
d. 1, 2 and 3

Answer: a
Explanation:
The following are the  Central Armed Police Forces of India, their deployment along India’s border is as follows:
Border Security Force (BSF)- Pakistan and Bangladesh
Indo-Tibetan Border Police (ITBP)-China
Sashastra Seema Bal (SSB)- Nepal and Bhutan

43. Consider the following statements:

1.      The Hunter Commission appointed to investigate the Jallianwala Bagh shootings did not have any Indian members.
2.      The Hunter Commission did not impose any penal or disciplinary action against General Dyer.

Which of the given statement/s is/are incorrect?

a. 1 only
b. 2 only
c. Both 1 and 2
d. Neither 1 nor 2

Answer: a
Explanation:
The government formed a committee of inquiry to investigate the Jallianwala Bagh shootings. On October 14, 1919, the Government of India announced the formation of the Disorders Inquiry Committee. The committee was commonly known as Hunter Commission after the name of its chairman, Lord William Hunter. It also had Indian members. However, the Hunter Committee did not impose any penal or disciplinary action against General Dyer.

Q44. Fair and Remunerative Prices (FRP) for Sugarcane is announced by?

a. Commission for Agricultural Costs and Prices (CACP)
b. Cabinet Committee on Economic Affairs (CCEA)
c. Department of Agriculture and Co-operation and Farmers Welfare
d. None of the above

Answer: b
Explanation:
Fair and remunerative price (FRP) is the minimum price at which rate sugarcane is to be purchased by sugar mills from farmers. On the recommendation of the Commission for Agricultural Costs and Prices (CACP), FRP is announced by the Cabinet Committee on Economic Affairs (CCEA), which is chaired by the Prime Minister.

Q45. Which of the following is/are not correctly matched?

1.      Tso Moriri – Manipur
2.      Deepor Beel – Assam
3.      Rudra Sagar Lake – Tripura

Choose the correct option:

a. 1 only
b. 1 and 3 only
c. 3 only
d. 2 and 3 only

Answer: a
Explanation:
Tso Moriri, Deepor Beel and Rudra Sagar Lake are all Ramsar sites in India. Tso Moriri is is a lake in the Changthang Plateau in Ladakh. Loktak lake is the Ramsar Site in the state of Manipur.

46. Teesta River flows through the states of:

1.      Sikkim
2.      West Bengal
3.      Meghalaya

Choose the correct option:

a. 1 only
b. 1 and 3 only
c. 1 and 2 only
d. 1, 2 and 3

Answer: c
Explanation:
Teesta is a 414km long river flowing through the Indian states of West Bengal, Sikkim before going to the Bay of Bengal through Bangladesh.

47. Indian Oil Corporation Ltd (IOC) recently launched a special winter-grade diesel for Ladakh. Consider the following statements

1.      It remains unfrozen up to minus 33 degree Celsius.
2.      It meets BIS specification of BS-VI grade diesel.

Select the correct statements

 a) 1 Only
 b) 2 Only
 c) Both
 d) None

Solution (c)
Indian Oil Corporation has developed winter-grade diesel for Ladakh to address the problem of loss of fluidity in fuel during extreme winter conditions.
The winter grade diesel produced by Panipat Refinery for the first time has a pour point of – 33oC and does not lose its fluidity function even in the extreme winter weather of the region unlike the normal grade of diesel which becomes exceedingly difficult to utilize.
This winter grade diesel also meets BIS specification of BS-VI grade diesel.

48. ‘India International Cherry Blossom Festival’ is celebrated in which of the following States?

 a) Uttarakhand
 b) Arunachal Pradesh
 c) Meghalaya
 d) Manipur

Solution (c)
The India International Cherry Blossom Festival-2019 is organised by the Government of Meghalaya in association with Institute of Bioresources and Sustainable Development (IBSD). They had planted 5000 cherry trees between Umiam Lake, the Wards’ Lake, New Shillong and all the way to Mawphlang. The target is to plant 20000 more trees.

49. ‘SAANS’ campaign is associated with

 a) Breast Feeding
 b) Pneumonia
 c) Rotavirus
 d) Menstrual Hygiene

Solution (b)
SAANS, short for ‘Social Awareness and Action to Neutralise Pneumonia Successfully’ was launched by the Union Health Ministry to mobilise people to protect children from pneumonia, and train health personnel and other stakeholders to provide prioritised treatment to control the disease.

50. Recently, two varieties of tea, Green Tea and White Tea received GI Tags. It belongs to which of the following States?

 a) West Bengal
 b) Assam
 c) Tamil Nadu
 d) Tripura

Solution (a)
Darjeeling green and white teas, whose price varies from Rs 3,000 -Rs 10,000 per kg, have received Geographical Indication (GI) tag for the domestic market, which will put to rest any doubt about their authenticity and quality.

51. ‘Sisseri River Bridge’ which was opened recently, is located in

 a) Ladakh
 b) Arunachal Pradesh
 c) Jammu & Kashmir
 d) Assam

Solution (b)
Defence Minister inaugurated the Sisseri River Bridge located at lower Dibang Valley in Arunachal Pradesh. The 200m long bridge provides connectivity between Dibang Valley and Siang.

52. ‘Kimberley Process Certification Scheme (KPCS)’ is associated with

 a) Diamonds
 b) Terbium
 c) Yttrium
 d) Uranium

Solution (a)
The Kimberley Process Certification Scheme (KPCS) is the process established in 2003 to prevent “conflict diamonds” from entering the mainstream rough diamond market by United Nations General Assembly Resolution 55/56 following recommendations in the Fowler Report.

53.‘Za’ir-Al-Bahr’ is a naval exercise between India and

 a) Oman
 b) Qatar
 c) Saudi Arabia
 d) United Arab Emirates

Solution (b)
A three-day naval exercise, Za’ir-Al-Bahr (Roar of the Sea) is being conducted between November 17 and November 21, 2019 between the Indian and Qatar Naval Forces.

54.The objectives of ‘NISHTHA’ mega-training programme are:

1.      To equip all the elementary stage teachers on learning outcomes, school-based assessment, learner –centered pedagogy, new initiatives in education and addressing diverse needs of children through multiple pedagogies.
2.      To orient state functionaries and school principals on learning outcomes, national achievement survey, learner- centered pedagogy and new initiatives in school education so that they are able to monitor schools and extend support to schools for the implementation of new initiatives.

Select the correct code:

 a) 1 Only
 b) 2 Only
 c) Both
 d) None

Solution (c)
NISHTHA – National Initiative for School Heads’ and Teachers’ Holistic Advancement is an initiative to build capacities of teachers and school principals at the elementary stage.
The initiative is an Integrated Teacher Training Programme of the Department of School Education and Literacy, Ministry of HRD as part of its National Mission to improve learning outcomes at the Elementary level under the Centrally Sponsored Scheme of Samagra Shiksha during 2019-20.
The programme aims to build the capacities of around 42 lakh participants covering all teachers and Heads of Schools at the elementary level in all Government schools, faculty members of State Councils of Educational Research and Training (SCERTs), District Institutes of Education and Training (DIETs) as well as Block Resource Coordinators and Cluster Resource Coordinators in all States and UTs.
The objectives of Nishtha mega-training programme are:
To equip all the elementary stage teachers on learning outcomes, school based assessment, learner –centred pedagogy, new initiatives in education and addressing diverse needs of children through multiple pedagogies, etc.
To monitor and provide extensive support to the teachers using multiple modes up to the classroom level, in view of improving learning outcomes of the students.
To orient state functionaries and school principlas on learning outcomes, national achievement survey, learner- centred pedagogy and new initiatives in school education so that they are able to monitor schools and extend support to schools for the implementation of new initiatives.

55. Consider the following statements with respect to the ‘Sun’

1.      The Core of the Sun is hotter than the Corona.
2.      The Corona is hotter than the Photosphere.

Select the correct statements

 a) 1 Only
 b) 2 Only
 c) Both
 d) None

Solution (c)
Core > Corona > Photosphere.
The temperature at the core of the Sun is nearly 15 million degrees Celsius, while that at its surface layer, known as the photosphere, is merely 5,700 degrees C. The natural thing to expect is that still further outwards, in its atmosphere, known as the corona, the temperatures would be comparable to that at the surface (photosphere). However, the temperature of the corona is much higher. It starts increasing outside the photosphere, reaching a value of about one million degrees or more in the corona.
Read More – https://www.thehindu.com/sci-tech/science/long-standing-conundrum-on-the-suns-atmosphere-solved/article29993252.ece

56. ‘Gravastar’ was in news recently. What is it associated with?

 a) Black Holes
 b) Jupiter’s Moon
 c) South Pole of the Moon
 d) Lithium Ion Batteries for Aviation

Solution (a)
A gravastar is an object hypothesized in astrophysics as an alternative to the black hole theory by Pawel O. Mazur and Emil Mottola. It has usual black hole metric outside of the horizon, but de Sitter metric inside. On the horizon there is a thin shell of matter. The term “gravastar” is a portmanteau of the words “gravitational vacuum star”.

57.Consider the following statements with respect to ‘Solar eclipse’

1.      A solar eclipse occurs when the moon passes in a direct line between the Earth and the sun.
2.      Solar Eclipse takes place twice a year in the month of May and November.

Select the correct statements

 a) 1 Only
 b) 2 Only
 c) Both
 d) None

Solution (a)
A solar eclipse happens when the Moon is directly in line between Earth and the Sun. The Moon’s shadow sweeps across Earth’s surface; at those places.
Total solar eclipses are rare events. Although they occur somewhere on Earth every 18 months on average, it is estimated that they recur at any given place only once every 360 to 410 years, on average.

58. Consider the following statements with respect to ‘Dam Rehabilitation and Improvement Project (DRIP)’

1.      It is a State Sector Scheme with a Central component
2.      It is funded by the World Bank

Select the correct statements

 a) 1 Only
 b) 2 Only
 c) Both
 d) None

Solution (c)
Both the statements are correct.

59. ‘Hynniewtrep National Liberation Council’ which was in news is associated with

 a) Meghalaya
 b) Tripura
 c) Nagaland
 d) Manipur

Solution (a)
The central government has banned ‘Hynniewtrep National Liberation Council’ (HNLC), a Meghalaya-based insurgent group, for its increased activities of violence and other insurgent acts. Centre also notified that HNLC’s activities are detrimental to the sovereignty and integrity of India.

60. Consider the following statements with respect to ‘Farmers’ Clubs (FCs)’

1.      The major objective is to promote “Development through credit, technology transfer, awareness and capacity building” of the farmers.
2.      It is promoted by National Bank for Agriculture and Rural Development (NABARD).

Select the correct statements

 a) 1 Only
 b) 2 Only
 c) Both
 d) None

Solution (c)
Farmers’ Clubs (FCs) promoted by National Bank for Agriculture and Rural Development (NABARD) are grass root level informal fora organized by the rural branches of banks, Non-Government Organisations (NGOs), Krishi Vigyan Kendras (KVKs), etc. The programme is being implemented for the mutual benefit of the banks and the farmers. The major objective is to promote “Development through credit, technology transfer, awareness and capacity building” of the farmers. The clubs are beneficial for banks as well as line departments of the State Government for convergence of the programmes / schemes sponsored / implemented by them.

61. Consider the following statements with respect to “Project 3-S: Smart Safety Surveillance”

1.      It is a project focused on optimizing pharmacovigilance (PV) systems in low-middle and middle-income countries (LMICs) across the globe.
2.      The World Health Organization (WHO) along with the Bill and Melinda Gates Foundation is behind this project.

Select the correct statements

 a) 1 Only
 b) 2 Only
 c) Both
 d) None

Solution (c)
Project 3-S: Smart Safety Surveillance
It is a project focused on optimizing pharmacovigilance (PV) systems in low-middle and middle-income countries (LMICs) across the globe by:
Developing an advocacy and communications strategy for the next 3 years.
Identifying, securing and promoting representation at prominent global health events and platforms where 3-S advocacy activities can be launched.
Generating supportive marketing materials, including publications
WHO has been prodding countries to adopt the 3S programme, with the support of the Bill and Melinda Gates Foundation, to strengthen pharmacovigilance systems in developing nations.
The central government will expand the reach of the Smart Safety Surveillance, or 3S, programme, to optimize post-marketing surveillance of priority drugs and vaccines, and ensure the vaccines distributed under the universal immunization programme are safe.
The 3S project was recommended by the World Health Organization (WHO), considering the limited safety data on vaccines introduced in India.
As part of the 3S project, India is evaluating the recently-introduced rotavirus vaccines. It is also trying to strengthen the collaboration among key stakeholders, such as ministry of health and Central Drugs Standard Control Organisation (CDSCO), to ensure high levels of vigilance.

62. ‘World Energy Outlook’ is published by

 a) World Economic Forum
 b) International Energy Agency
 c) International Renewable Energy Agency
 d) International Energy Forum

Solution (b)

63. The Union Cabinet has given its approval for introduction of the Industrial Relations Code, 2019 in the Parliament. The code on Industrial Relations has been prepared after amalgamating, simplifying and rationalizing the provisions of which of the following Acts?

1.      The Trade Unions Act, 1926
2.      Minimum Wages Act 1948
3.      The Industrial Disputes Act, 1947

Select the correct code:

 a) 1 and 2
 b) 2 and 3
 c) 1 and 3
 d) All of the above

Solution (c)
The draft code on Industrial Relations has been prepared after amalgamating, simplifying and rationalizing the relevant provisions of following three Central Labour Acts:
The Trade Unions Act, 1926
The Industrial Employment (Standing Orders) Act, 1946
The Industrial Disputes Act, 1947

64. The CT-TTX, Counter Terrorism Table Top Exercise was in news recently. It is associated with

 a) SCO
 b) QUAD
 c) BRICS
 d) ASEAN

Solution (b)

65. ‘Global Terrorism Index’ is published by

 a) Institute for Economics and Peace
 b) United Nations Security Council
 c) Institute of Peace and Conflict Studies
 d) International Criminal Police Organization

Solution (a)
The Global Terrorism Index (GTI) is a report published annually by the Institute for Economics and Peace (IEP), and was developed by IT entrepreneur and IEP’s founder Steve Killelea.

66. Consider the following statements with respect to ‘’Sowa Rigpa’ medicine system

1.      It was introduced in India by the Arabs and Persians sometime around the eleventh century.
2.      The foundation of the medicine system was laid by Hippocrates.

Select the correct statements

 a) 1 Only
 b) 2 Only
 c) Both
 d) None

Solution (d)
“Sowa-Rigpa” commonly known as Amchi system of medicine is one of the oldest, Living and well documented medical tradition of the world. It has been popularly practice in Tibet, Magnolia, Bhutan, some parts of China, Nepal, Himalayan regions of India and few parts of former Soviet Union etc. There are various schools of thought about the origin of this medical tradition, some scholars believe that it is originated from India; some says China and others consider it to be originated from Tibet itself. The majority of theory and practice of Sowa-Rigpa is similar to “Ayurveda”. The first Ayurvedic influence came to Tibet during 3rd century AD but it became popular only after 7th centuries with the approach of Buddhism to Tibet. There after this trend of exportation of Indian medical literature, along with Buddhism and other Indian art and sciences were continued till early 19th century.

67. ‘Vientiane Action Programme’ is associated with

 a) APEC
 b) ASEAN
 c) BIMSTEC
 d) RCEP

Solution (b)
Vientiane Action Programme calls for ASEAN to build a peaceful, secure and prosperous ASEAN.

68. Signal Fish which was in news recently was discovered in?

a.  Andaman and Nicobar Islands
b.  Gulf of Khambhat
c.  Kerala coast
d.  Gulf of Mannar


Answer : c
India's first Signal Fish was found in Kerala deep waters.
The discovered species is the world’s largest Signal Fish.
The species named Pteropsaron indicum was collected by trawlers at a depth of 70 metres during a marine biodiversity survey.
The signal fishes are spotted in the coral reefs. So it indicates the presence of patchy corals off the Kerala coast.

69.  Which of the following are the goals of the Curiosity Rover?

1.      Determine whether life ever arose on Mars.
2.      Characterize the climate of Mars.
3.      Characterize the geology of Mars.
4.      Prepare for human exploration.

Select the correct answer using the codes given below:

a.  1 and 2 only
b.  1, 2 and 4 only
c.  1, 2 and 3 only
d.  1, 2, 3 and 4

Answer : d
According to the new study, Oxygen behaves in a mysterious way on Mars.
NASA’s Curiosity rover’s has added a new mystery for scientists studying Mars, with samples from the rover showing a seasonal change around Oxygen levels near the surface of the Gale Crater on the red planet.
NASA found that Oxygen levels throughout spring and summer rose by as much as 30%.
According to NASA, Curiosity has 4 main science goals in support of the agency's Mars exploration program:
Determine whether life ever arose on Mars.
Characterize the climate of Mars.
Characterize the geology of Mars.
Prepare for human exploration.

70. Consider the following statements with respect to Places of Worship Act, 1991

1.      The aim of the Act was to freeze the status of any place of worship as it existed on August 15, 1947.
2.      The Act does not apply to the place of worship commonly referred to as Ram Janmabhoomi-Babri Masjid in Ayodhya.
3.      This law will have overriding effect over any other law in force.
Which of the statement(s) given above is/are correct?

a.  3 only
b.  1 and 2 only
c.  1, 2 and 3
d.  None

Answer : c
The Act declares that the religious character of a place of worship shall continue to be the same as it was on August 15, 1947.
It says no person shall convert any place of worship of any religious denomination into one of a different denomination or section.
It was intended to pre-empt new claims by any group about the past status of any place of worship and attempts to reclaim the structures or the land on which they stood.
It declares that all suits, appeals or any other proceedings regarding converting the character of a place of worship, which are pending before any court or authority on August 15, 1947, will abate as soon as the law comes into force.
No further legal proceedings can be instituted.
However, there is an exception to the bar on instituting fresh proceedings with regard to suits that related to conversion of status that happened after August 15, 1947.
This saves legal proceedings, suits and appeals regarding chance of status that took place after the cut-off date.
These provisions will not apply to
ancient and historical monuments and archaeological sites and remains that are covered by the Ancient Monuments and Archaeological Sites and Remains Act, 1958
a suit that has been finally settled or disposed of and
any dispute that has been settled by the parties or conversion of any place that took place by acquiescence before the Act commenced.
The Act does not apply to the place of worship commonly referred to as Ram Janmabhoomi-Babri Masjid in Ayodhya.
This law will have overriding effect over any other law in force, it said.

71. Consider the following statements with respect to Bharatiya Poshan Krishi Kosh

1.      It is a repository of diverse crops across 128 agro-climatic zones for better nutritional outcomes.
2.      It aims at reducing malnutrition through a multi-sectoral results-based framework, including agriculture, among women and children across the country. 
3.      Ministry of Women and Child Development has collaborated with Bill and Melinda Gates Foundation for this project.

 Which of the statement(s) given above is/are correct?

a.  1 only
b.  1 and 2 only
c.  1 and 3 only
d.  1, 2 and 3

Answer : d
Union Government has recently announced Bharatiya Poshan Krishi Kosh which is a repository of diverse crops across 128 agro-climatic zones for better nutritional outcomes.
The Kosh aims at reducing malnutrition through a multi-sectoral results-based framework, including agriculture, among women and children across the country.
Ministry of Women and Child Development has collaborated with Bill and Melinda Gates Foundation for this project.
The Bharatiya Poshan Krishi Kosh seeks to promote healthy dietary practices and tackle under-nutrition in a sustainable manner.
The initiative will be built on an essential understanding of social, behavioural and cultural practices that promote and reinforce healthy dietary behaviours both at the individual and community level.

72. Consider the following statements with respect to Health Systems for a New India: Building Blocks Report which was released recently:

1.      It was released recently by the Ministry of Health and Family Welfare.
2.      The report has out rightly suggested that Pradhan Mantri Jan Arogya Yojana (PM-JAY) should be extended to whole of India.

Which of the statement(s) given above is/are correct?

a.  1 only
b.  2 only
c.  Both 1 and 2
d.  Neither 1 nor 2

Answer : d
Government think tank Niti Aayog recently released a report ‘Health Systems for a New India: Building Blocks—Potential Pathways to Reforms’.
The report identified five focus areas of future health system - to deliver on unfinished public health agenda, change health financing away from out of pocket so spend into large insurers, integrate service delivery vertically and horizontally, empower citizens to become better buyers of health, harness the power of digital health.
Even as the report has not out rightly suggested that Pradhan Mantri Jan Arogya Yojana (PM-JAY), government’s cashless health insurance scheme covering 10 crore poor families for Rs five lakh annually, should be extended to whole of India, it discretely mentioned that PM-JAY should be considered with an eye on its potential to influence the overall healthcare transformation in India, beyond its current explicit mandate.

73.CASE Instrument sometimes seen in the news recently was contributed by _________ for _________ Mission?

a.  NASA – ESA
b.  ESA – NASA
c.  NASA – ROSCOSMOS
d.  JAXA – ROSCOSMOS

Answer : a
NASA has announced that it will contribute an instrument to a European space mission that will explore the atmospheres of planets or exo-planets orbiting stars beyond our Sun for the first time.
The instrument will aid the European Space Agency’s (ESA’s) ARIEL or Atmospheric Remote-sensing Infrared Exoplanet Large-survey mission.
NASA’s instrument has been called the CASE or Contribution to ARIEL Spectroscopy of Exoplanets and it will be managed by NASA’s Jet Propulsion Laboratory in Pasadena, California.
The ARIEL spacecraft with CASE on board is expected to launch in 2028.

74. Consider the following statements

1.      He is also called as Punjab Kesari.
2.      His important works include ‘The Arya Samaj’, ‘Young India’, and ‘England’s Debt to India’.
3.      He died at Lahore in 1928 after he was attacked by police during a protest rally against the Simon Commission.

Identify the personality who correctly matches with the above description:

a.  Bal Gangadhar Tilak
b.  Bipin Chandra Pal
c.  Lala Lajpat Rai
d.  Abul Kalam Azad

Answer : c
Lala Lajpat Rai, the firebrand Indian nationalist leader was affectionately called as ‘Punjab Kesari’.
Rai is remembered for his role during the Swadeshi movement and for his advocacy of education.
The patriot died at Lahore in 1928 after he was attacked by police during a protest rally against the Simon Commission.
His important works include: ‘The Arya Samaj’, ‘Young India’, ‘England’s Debt to India’, ‘Evolution of Japan’, ‘India’s Will to Freedom’, ‘Message of the Bhagwad Gita’, ‘Political Future of India’, ‘Problem of National Education in India’, ‘The Depressed Glasses’, and the travelogue ‘United States of America’.

75. Consider the following statements with respect to Diesel

1.      Below the Cloud point, Diesel is prone to waxing.
2.      A special winter-grade diesel with a low pour-point of -33 deg C, does not lose its fluidity even in extreme winter conditions.

Which of the statement(s) given above is/are correct?

a.  1 only
b.  2 only
c.  Both 1 and 2
d.  Neither 1 nor 2

Answer : c
Recently, 1st Winter-Grade Diesel outlet was inaugurated in Ladakh.
The move will boost tourism in extreme weather conditions and help in the economic development of the region.
Under normal conditions the paraffin wax added to diesel improves viscosity and lubrication.
However, when temperatures begin to fall, this paraffin wax thickens and turns into a cloudy mixture. This point is known as a fuel’s ‘Cloud point’.
Indian Oil has come up with an innovative solution to this problem by introducing a special Winter-grade diesel with a low pour-point of -33 deg Celsius, which does not lose its fluidity function even in extreme winter conditions.
This winter grade diesel also meets BIS specification of BS-VI grade diesel.

76) Consider the following statements with respect to  Epilepsy

1.      It is a chronic communicable disease of the brain that affects people of all ages.
2.      It is seen as a major health problem and it is considered to be a Mental health disorder.

Which of the statement(s) given above is/are correct?

a.  1 only
b.  2 only
c.  Both 1 and 2
d.  Neither 1 nor 2

Answer : d
In India, November 17th is observed every year as National Epilepsy Day to create awareness about epilepsy.
It is a chronic noncommunicable disease of the brain that affects people of all ages.
It is seen as a major health problem that was earlier considered to be a ‘Mental health disorder’, however, with advancement in diagnosis and epidemiological studies, epilepsy is now considered a ‘Neurological disorder’.
Around 50 million people worldwide have epilepsy, making it one of the most common neurological diseases globally.
Nearly 80% of people with epilepsy live in low- and middle-income countries.
It is estimated that up to 70% of people living with epilepsy could live seizure- free if properly diagnosed and treated.
In many parts of the world, people with epilepsy and their families suffer from stigma and discrimination.

77) ‘Nadu-Nedu’ scheme which was in news recently was launched in

a.  Andhra Pradesh
b.  Kerala
c.  Telangana
d.  Karnataka

Answer : a
Andhra Pradesh government has launched the ‘Nadu-Nedu’ programme.
The programme aims to introduce English medium from classes 1 to 6 in state-run schools.
The scheme will be implemented in 15,715 schools in the first phase.
It will cover all schools in three years’ time with a budget of Rs 12,000 crores.
This  will transform all government schools with the required infrastructure and upgradation of skills besides setting up of English labs in addition to providing basic amenities like clean environs, furniture, toilets, running water, compound wall, blackboard, renovation and involve parent committee.

78. Consider the following statements with respect to ASEAN Defence Ministers’ Meeting-Plus (ADMM-Plus)

1.      ADMM-Plus is a platform comprising ASEAN and its eight dialogue partners including India and United States.
2.      The grouping focuses its work on maritime security, counter-terrorism, humanitarian assistance and cyber Security.
3.      India is not participating in the 6th edition of ADMM-Plus in response to the failure in reaching consensus for Regional Comprehensive Economic Partnership (RCEP) Agreement.

Which of the statement(s) given above is/are correct?

a.  2 only
b.  1 and 2 only
c.  2 and 3 only
d.  1, 2 and 3

Answer : b
Consistent with the ADMM guiding principles of open and outward looking, the 2ndADMM in Singapore in 2007 adopted the Concept Paper to establish the ADMM-Plus.
The ADMM-Plus is a platform for ASEAN and its eight Dialogue Partners to strengthen security and defence cooperation for peace, stability, and development in the region.
The ADMM-Plus countries include ten ASEAN Member States, namely, Brunei Darussalam, Cambodia, Indonesia, Lao PDR, Malaysia, Myanmar, the Philippines, Singapore, Thailand and Viet Nam, and eight Plus countries, namely Australia, China, India, Japan, New Zealand, ROK, Russian Federation and the United States.
The Inaugural ADMM-Plus was convened in Ha Noi, Viet Nam, on 12 October 2010.
The Defence Ministers then agreed on five areas of practical cooperation to pursue under this new mechanism, namely maritime security, counter-terrorism, humanitarian assistance and disaster relief, peacekeeping operations and military medicine.
To facilitate cooperation on these areas, Experts' Working Groups (EWGs) are established.
In 2013 and 2016, new priority areas like humanitarian mine action and cyber security were agreed respectively.
Union Minister for Home Affairs recently attended the 6th ASEAN Defence Ministers’ Meeting – Plus.

79. Consider the following statements with respect to Kimberley Process Certification Scheme (KPCS)

1.      Kimberley Process is a joint initiative involving Government, International Diamond Industry and Civil Society to stem the flow of Rough Diamonds.
2.      The Chair of Kimberley Process is selected annually based on election among the member countries.
3.      India is one of the founder members of Kimberley Process Certification Scheme and is the Chair of Kimberley Process for the year 2019.

Which of the statement(s) given above is/are correct?

a.  3 only
b.  1 and 3 only
c.  2 and 3 only
d.  None

Answer : a
The Kimberley Process is a joint initiative involving Government, International Diamond Industry and Civil Society to stem the flow of Conflict Diamonds.
“Conflict Diamonds” means rough diamonds used by rebel movements or their allies to finance conflict aimed at undermining legitimate governments.
It is also described in United Nations Security Council (UNSC) resolutions.
At present, KPCS has 55 members representing 82 countries including EU with 28 members.
The Kimberley Process is chaired, on a rotating basis, by participating countries.
KP Vice-Chair is generally elected by KP Plenary each year who becomes the Chair in the next year.
India is the current Chair of KPCS with Russian Federation as Vice Chair for the year 2019.
Since 2003, India has been actively participating in the KPCS process and is a member of all Working Groups of KP (except for WGAAP).
Department of Commerce is the nodal Department and Gem & Jewellery Export Promotion Council (GJEPC) is designated as the KPCS Importing and Exporting Authority in India.
GJEPC is responsible for issuing KP Certificates and is also the custodian of KP Certificates received in the country.

80. Consider the following statements with respect to National Mission on Cultural Mapping (NMCM)

1.      The Mission will compile data of artists, art forms & geo location with inputs from Central Ministries, State Governments & art/culture bodies.
2.      It has been set up by the Ministry of Culture in 2017.

Which of the statement(s) given above is/are correct?

a.  1 only
b.  2 only
c.  Both 1 and 2
d.  Neither 1 nor 2

Answer : c
National Mission on Cultural Mapping (NMCM) has been set up by the Ministry of Culture in 2017.
Mission will compile data of artists, art forms & geo location with inputs from Central Ministries, State Governments & art/culture bodies.
Specially designed data capture form with technical collaboration of National E-Governance Division (NEGD)/Ministry of Electronics and Information Technology (MEITY) has been formulated for data collection.

81. Consider the following statements with respect to Exercise Za’ir-Al-Bahr

1.      It is a multilateral maritime exercise held between members of Gulf Cooperation Council (GCC).
2.      For the first time, India and China is participating in this exercise as special observers.

Which of the statement(s) given above is/are correct?

a.  1 only
b.  2 only
c.  Both 1 and 2
d.  Neither 1 nor 2

Answer : d
Navies of India and Qatar have started a five-day bilateral maritime exercise Za'ir-Al-Bahr (Roar of the Sea) at Doha that includes surface action, air defence, maritime surveillance as well as social and sports events.
The inaugural edition of the bilateral maritime exercise would further strengthen the robust defence co-operation between the two countries, especially in the fight against terrorism, maritime piracy and security.

82. Consider the following statements with respect to Starlink satellites which was in news recently

1.      It aimed at providing low-cost and reliable space-based Internet services to the world.
2.      It was launched by the European Space Agency (ESA)

Which of the statement(s) given above is/are correct?

a.  1 only
b.  2 only
c.  Both 1 and 2
d.  Neither 1 nor 2

Answer : a
A spray of 60 Starlink satellites fired into orbit, the first operational batch of what is intended to eventually evolve into a constellation of nearly 12,000 satellites.
It aimed at providing low-cost and reliable space-based Internet services to the world.
It was launched by SpaceX, the world’s leading private company in space technology.
But astronomers worry the SpaceX satellites are too bright and could form a "megaconstellation" that blots out the star and interferes with the work of telescopes.
Astronomers and scientists have also complained about increased “light-pollution”.

83. Consider the following statements with respect to Pradhan Mantri Matru Vandana Yojana (PMMVY)

1.      It is to support lactating mothers and pregnant women by compensating them for loss of wages during their pregnancy.
2.      It is targeted only at women delivering their first child.

Which of the statement(s) given above is/are correct?

a.  1 only
b.  2 only
c.  Both 1 and 2
d.  Neither 1 nor 2

Answer : c
A recent survey found that around Maternity scheme reaches only one-third of beneficiaries.
The exercise, titled Jaccha Baccha-Survey (JABS), is critical of the Pradhan Mantri Mantru Vandana Yojana (PMMVY), launched in 2017 as part of obligations under the National Food Security Act.
Researchers assert that extrapolation of RTI data show only 31% of eligible mothers got benefits.
Almost 63% of pregnant women in rural India work right until the day of delivery.
And, 49% say they felt exhausted during pregnancy, due to lack of food and rest, found in the survey.
Pradhan Mantri Matru Vandana Yojana (PMMVY) is a vital programme to support lactating mothers and pregnant women by compensating them for loss of wages during their pregnancy.
The PMMVY is targeted only at women delivering their first child.
A cash amount of Rs. 6,000 is transferred to the bank account of the beneficiary in 3 instalments upon meeting certain conditions including early registration of pregnancy, having at least one ante-natal check-up and registration of child birth.

84. Which of the following countries have a coastline on the Barents Sea?

1.      Norway
2.      Finland
3.      Russia
4.      Sweden

Choose the correct option from the option given below

a.  1 and 2 only
b.  1 and 3 only
c.  2 and 3 only
d.  1, 2 and 4 only

Answer : b
Recently, it was found that King Crabs, which are not native to Barents sea, have slowly conquered the region ever since their introduction.
Climate change isalso making it easier to fish the crabs, with much of the Barents ice-free for longer months.
The Barents Sea is classified as a ‘Marginal sea of the Arctic Ocean’.
Climatic changes have led to the Barents Sea having characteristics similar to the Atlantic Ocean.
The Barents Sea is off the northern coasts of Russia and Norway.
The Russian and Norwegian governments were involved in a border dispute over the position of their respective border in the sea.

85. Consider the following statements with respect to Agni II

1.      It is a surface-to-surface medium range nuclear-capable missile.
2.      It is developed by the Defence Research and Development Organisation (DRDO).

Which of the statement(s) given above is/are correct?

a.  1 only
b.  2 only
c.  Both 1 and 2
d.  Neither 1 nor 2

Answer : c
India on Saturday successfully conducted the first, night trial of Agni-II from the Abdul Kalam Island off the Odisha coast.
It is a surface-to-surface medium range nuclear-capable missile.
Agni-II, an intermediate range ballistic missile (IRBM), has already been inducted into the armed forces.
Agni-II was developed by Advanced Systems Laboratory along with other DRDO laboratories and integrated by the Bharat Dynamics Limited.

86. Consider the following statements with respect to World Gold Council

1.      It is an association under International Council on Mining and Metals (ICMM).
2.      Its members includes most of the world’s largest gold mining companies.

Which of the statement(s) given above is/are correct?

a.  1 only
b.  2 only
c.  Both 1 and 2
d.  Neither 1 nor 2

Answer : b
Gold remains the third-most popular investment choice, with 46% of all global retail investors choosing gold products, next only to saving accounts (78%) and life insurance (54%), reveals a new World Gold Council (WGC) survey of retail consumers.
The World Gold Council is the market development organisation for the gold industry.
It is a nonprofit association of the world's leading gold producers and it does not come under ICMM.
Its purpose is to stimulate and sustain demand for gold, provide industry leadership, and be the global authority on the gold market.
Its members includes most of the world’s largest gold mining companies.
It is based in the UK, with operations in India, China, Singapore and the US , and it is the lead for the gold industry worldwide.

87. Consider the following statements with respect to Arundhati scheme

1.      Under the Scheme, government of West Bengal provides 1 Tola Gold to Brides parents.
2.      The scheme is limited only to the weaker section of the society whose annual income is below Rs 5 lakh irrespective of caste, creed and religion.

Which of the statement(s) given above is/are correct?

a.  1 only
b.  2 only
c.  Both 1 and 2
d.  Neither 1 nor 2

Answer : b
In a unique initiative to check child marriage, Assam Govt approves Arundhati scheme to provide 1 Tola Gold (11.66 grams) to Brides parents belonging to the economically weaker sections of the society.
With the aim to reduce child marriage under the Prohibition of Child Marriage Act that a girl in India can’t get married before the age of 18, and a boy can’t get married before 21 years of age, this scheme can be availed by formal registration of the marriage under the Special Marriage (Assam) Rules, 1954 at the time of marriage.
This is only limited to the weaker section of the society whose annual income is below Rs 5 lakh irrespective of caste, creed, religion etc.

88. Consider the following statements with respect to End-to-end encryption Technology

1.      It scrambles messages in such a way that they can be deciphered only by the sender and the intended recipient.
2.      It steers governments away from mass surveillance and toward a more targeted, constitutional form of intelligence gathering.
3.      Law enforcement agencies can get access to messages encrypted through this technology by presenting technology companies with a warrant.

Which of the statement(s) given above is/are correct?

a.  2 only
b.  1 and 2 only
c.  1 and 3 only
d.  1, 2 and 3

Answer : b
End-to-end encryption Technology

How it Works?

It scrambles messages in such a way that they can be deciphered only by the sender and the intended recipient.
As the label implies, end-to-end encryption takes place on either end of a communication.
A message is encrypted on a sender’s device, sent to the recipient’s device in an unreadable format, then decoded for the recipient.
There are several ways to do this, but the most popular works like this: A program on your device mathematically generates two cryptographic keys — a public key and a private key.
The public key can be shared with anyone who wants to encrypt a message to you.
The private key, or secret key, decrypts messages sent to you and never leaves your device.
Think of it as a locked mailbox. Anyone with a public key can put something in your box and lock it, but only you have the private key to unlock it.
How is it different from other forms of encryption?
A more common form of encryption, known as transport layer encryption, relies on a third party, like a tech company, to encrypt messages as they move across the web.
With this type of encryption, law enforcement and intelligence agencies can get access to encrypted messages by presenting technology companies with a warrant or national security letter. The sender and recipient would not have to know about it.
End-to-end encryption ensures that no one can eavesdrop on the contents of a message while it is in transit.
It forces spies or snoops to go directly to the sender or recipient to read the content of the encrypted message.
Or they must hack directly into the sender’s or recipient’s device, something that can be harder to do “at scale” and makes mass surveillance much more difficult.
Privacy activists, libertarians, security experts and human rights activists argue that end-to-end encryption steers governments away from mass surveillance and toward a more targeted, constitutional form of intelligence gathering.
But intelligence and law enforcement agencies argue that end-to-end encryption makes it much harder to track terrorists, pedophiles and human traffickers.

89. Consider the following statements with respect to Mission Parivar Vikas

1.      It focus only on districts having the highest total fertility rates in the country.
2.      The main objective is to accelerate access to high quality family planning choices based on information, reliable services and supplies within a rights-based framework.

Which of the statement(s) given above is/are not correct?

a.  1 only
b.  2 only
c.  Both 1 and 2
d.  Neither 1 nor 2

Answer : d
The Ministry of Health and Family Welfare launched “Mission Parivar Vikas” in 145 high focus districts having the highest total fertility rates in the country.
These 145 districts are in the seven high focus, high TFR states of Uttar Pradesh, Bihar, Rajasthan, Madhya Pradesh, Chhattisgarh, Jharkhand and Assam that constitute 44% of the country’s population.
The main objective of ‘Mission Parivas Vikas’ will be to accelerate access to high quality family planning choices based on information, reliable services and supplies within a rights-based framework.
These 145 districts have been identified based on total fertility rate and service delivery (PPIUCD and Sterilization performance) for immediate, special and accelerated efforts to reach the replacement level fertility goals of 2.1 by 2025.

90. ‘SAANS Campaign’ which was in news recently is related to

a.  To reduce child mortality due to pneumonia
b.  An anti-tobacco awareness campaign
c.  To create awareness among consumers
d.  Cancer Awareness Campaign

Answer : a
Union Minister for Health and Family Welfare has launched SAANS Campaign which stands for ‘Social Awareness and Action to Neutralise Pneumonia Successfully’.
The aim of the campaign is to reduce child mortality due to pneumonia, which contributes to around 15% deaths annually of children under the age of five.
The campaign was launched in Gandhinagar, Gujarat at the 6th National Summit on Good, Replicable Practices and Innovations.

91.  Consider the following statements with respect to Central Drugs Standard Control Organisation (CDSCO)

1.      CDSCO under the Ministry of chemicals and fertilizers regulates the safety, efficacy and quality of notified medical devices.
2.      Drug Controller General of India (DCGI) an organ of CDSCO is responsible for approving and licensing of drugs and medical devices.

Which of the statement(s) given above is/are correct?

a.  1 only
b.  2 only
c.  Both 1 and 2
d.  Neither 1 nor 2

Answer : b
CDSCO is the national regulatory body under the Ministry of Health and Family Welfare for pharmaceuticals and medical devices in India.
It functions under the Drugs and Cosmetics Act.
CDSCO has six zonal offices, four sub-zonal offices, 13 port offices and seven laboratories under its control.
Major functions of CDSCO:
Regulatory control over the import of drugs
Approval of new drugs and clinical trials
Meetings of Drugs Consultative Committee (DCC) 
Drugs Technical Advisory Board (DTAB),
Approval of certain licenses as Central License Approving Authority is exercised by the CDSCO headquarters.
Drug Controller General of India (DCGI) is an organ of the CDSCO which is responsible for approving and licensing of drugs and medical devices.
The DCGI is advised by the Drug Technical Advisory Board (DTAB) and the Drug Consultative Committee (DCC).

92. Consider the following statements with respect to Tuberculosis (TB)

1.      The National Strategic Plan aims to end TB by 2030 in India inline with the global target.
2.      NI-KSHAY functions as the National TB Surveillance System and enables reporting of various surveillance data both in the public and private sector to the Government of India.

Which of the statement(s) given above is/are correct?

a.  1 only
b.  2 only
c.  3 only
d.  None

Answer : b
The National Strategic Plan 2017-25 for ending TB by 2025 was developed in 2017.
India’s target for complete elimination of Tuberculosis (TB) by 2025, is five years ahead of the global target of 2030.
NI-KSHAY-(Ni=End, Kshay=TB) is the web enabled patient management system for TB control under the Revised National Tuberculosis Programme (RNTCP).
It is developed and maintained by the Central TB Division (CTD), Ministry of Health and Family Welfare, in collaboration with the National Informatics Centre (NIC), and the World Health Organization Country office for India.
It also functions as the National TB Surveillance System and enables reporting of various surveillance data to the Government of India.
It is used by health functionaries at various levels across the country both in the public and private sector, to register cases under their care, record treatment details, monitor treatment adherence and to transfer cases between care providers.

93. Consider the following statements with respect to Greenhouse gas emissions

1.      Freshwater bodies do not emit Greenhouse gasses.
2.      Increasing levels of organic molecules can enhance microbial activity and produce more greenhouse gases.

Which of the statement(s) given above is/are correct?

a.  1 only
b.  2 only
c.  Both 1 and 2
d.  Neither 1 nor 2

Answer : b
Researchers from the University of Cambridge in the UK noted that every drop of fresh water contains thousands of different organic molecules that have previously gone unnoticed.
The study found that Climate change may cause the levels of greenhouse gases emitted by freshwater lakes to increase by between 1.5 and 2.7 times.
Freshwater ecosystems constitute a small fraction of our planet but play a disproportionately large and critical role in the global carbon cycle.
The amount of carbon that inland waters emit is comparable to the net amount of carbon absorbed by living organisms on Earth’s land surface and in its oceans.
Organic molecules are a food source for microbes in the lake sediments, which break them down and release carbon dioxide and methane as by-products.
Increasing levels of organic molecules can therefore enhance microbial activity and produce more greenhouse gases.
Since the same microbes can make greenhouse gases from many different organic molecules, the diversity of organic molecules was shown to be more closely linked with levels of greenhouse gas concentrations than the diversity of the microbes.
An elevated diversity of organic molecules may elevate greenhouse gas concentrations in waters because there are more molecules that can be broken down by sunlight penetrating the water.

94. In Modern World History, Velvet Revolution refers to which of the following?

b.  Modernization efforts in Iran
c.  Abolition of slavery in Haiti
d.  Fall of communist control over Czechoslovakia

Answer : d
With the so-called Velvet Revolution of 1989, Czechoslovakia freed itself of communist control and set out to adapt its command economy to the free market.
The government introduced a program based on policies of price liberalization, the opening of markets to foreign trade and investment, internal convertibility of the country’s currency, privatization of state-owned enterprises, and tax reform.
30th anniversary of the Velvet Revolution was held recently.

95 Apart from Farm fires which of the following is/are the main reasons for Delhi’s poor air quality?

Wind Speed
Wind direction
Geographic location

Select the correct answer using the codes given below:

a.  1 and 2 only
b.  1 and 3 only
c.  2 and 3 only
d.  1, 2 and 3

Answer : d
Wind direction and Speed – According to officials at the Central Pollution Control Board, a change in wind direction and a sudden increase in wind velocity were the main reasons behind Delhi going from severe to poor in a little over a day.
Wind blowing northwest brings pollution from Punjab and Haryana to Delhi. Later, the direction changed to northerly this improves the air quality.
The average wind speed in Delhi was under 10 km/hr. Because of an active western disturbance, the wind speed gradually rose and it was around 12 km/hr later due to which the air quality improved.
At greater speeds, pollutants far away can reach a spot within few hours and also be flushed out as quickly if the wind speed holds.
Geographic location – Delhi’s location — on the Indo-Gangetic Plain (IGP), the most polluted belt in the country is also one of the main reasons behind Delhi’s Air Pollution.

96 Consider the following statements with respect to Plague

1.      Both Pneumonic and Bubonic plague were caused by the Rattus Virus.
2.      Septicaemic plague is a rarer variant of plague which infects the lungs.

Which of the statement(s) given above is/are correct?

a.  1 only
b.  2 only
c.  Both 1 and 2
d.  Neither 1 nor 2

Answer : d
Pneumonic plague and Bubonic plague were types of plague caused by the Yersinia pestis bacterium.
Bites from infected fleas are the most common cause of bubonic plague infection, but the pneumonic variant — where the bacterium is breathed into the lungs — is more dangerous because it is spread through coughing.
A rarer third variant of the diseases is Septicaemic Plague, which infects the bloodstream.

97. Consider the following statements with respect to Hong Kong International Convention sometimes seen in the news recently:

1.      It was adopted by UN Environment for the Safe and Environmentally Sound Recycling of Ships in 2009.
2.      India has both signed and ratified the convention.

Which of the statement(s) given above is/are correct?

a.  1 only
b.  2 only
c.  Both 1 and 2
d.  Neither 1 nor 2

Answer : d
The International Maritime Organisation (IMO) adopted the Hong Kong International Convention for the Safe and Environmentally Sound Recycling of Ships in 2009.
It aimed at ensuring that ships being recycled after reaching the end of their operational lives do not pose any unnecessary risks to human health, safety and to the environment.
Cabinet has recently approved the proposal for enactment of Recycling of Ships Bill, 2019 and accession to the Hong Kong International Convention for Safe and Environmentally Sound Recycling of Ships, 2009.
The move will help India double its ship-recycling capacity over the next five years.
Germany became the 13th country to accede to the convention. One of the conditions for the convention to come into force is that at least 15 countries ratify it.
Background

India is the leader in the global ship recycling industry, with a share of over 30% of the market.
As per UNCTAD report on Review of Maritime Transport, 2018, India had demolished 6323 tonnes in 2017, of known ship scrapping across the world.
The ship-recycling industry is a labour-intensive sector, but it is susceptible to concerns on environmental safety.

98. Consider the following statements with respect to India – Europe 29 Business Forum

1.      The Europe 29 (E29) region refers to the 29 countries in Central, Eastern and Northern Europe.
2.      It has been organized by the Ministry of External Affairs and Confederation of Indian Industry (CII).

Which of the statement(s) given above is/are correct?

a.  1 only
b.  2 only
c.  Both 1 and 2
d.  Neither 1 nor 2

Answer : c
India – Europe 29 Business Forum has been recently organized by the Ministry of External Affairs, Government of India and Confederation of Indian Industry (CII) in New Delhi.
The Europe 29 (E29) region refers to the 29 countries in Central, Eastern and Northern Europe.
The India - Europe 29 Business Forum will act as a bridge and help close the geographical and knowledge gap between the two regions.
The Europe 29 region comprises Albania, Liechtenstein, Austria, Lithuania, Bosnia & Herzegovina, Macedonia, Bulgaria, Malta, Croatia, Moldova, Cyprus, Montenegro, Czech Republic, Norway, Denmark, Poland, Estonia, Romania, Finland, Serbia, Greece, Slovak Republic, Hungary, Slovenia, Iceland, Sweden, Latvia, Switzerland and Turkey.

99. Consider the following statements with respect to National Action Plan for Drug Demand Reduction (NAPDDR)

1.      It is an action plan prepared for the period from 2020-2025.
2.      It aims at reduction of adverse consequences of drug abuse through education, de-addiction and rehabilitation of affected individuals and their families.
3.      It was drafted by the Ministry of Health and Family Welfare.

Which of the statement(s) given above is/are correct?

a.  2 only
b.  1 and 2 only
c.  2 and 3 only
d.  1, 2 and 3

Answer : a
National Action Plan for Drug Demand Reduction (NAPDDR) has been prepared for 2018-2023.
It has been drafted by the Ministry of Social Justice and Empowerment.
It aims at reduction of adverse consequences of drug abuse through a multi-pronged strategy involving education, de-addiction and rehabilitation of affected individuals and their families.

100. IMD World Talent Ranking, 2019 was released recently by which of the following?

a.  INSEAD
b.  World Economic Forum
c.  International Institute for Management Development
d.  Both A and C

Answer : c
According to the IMD World Talent Ranking, 2019, India has slipped 6 places to 59 rank on the global annual list of 63 countries.
It was released by International Institute for Management Development (IMD).
The ranking is based on the performance in three main categories -- investment and development, appeal and readiness.
Image result for upsc images"